double sigma in complex numberIf complex no. ($z$) satisfying $frac12leq |z|leq 4;,$ Then Max. and Min. of $left|z+frac1zright|$complex no. $z$ such that $ |z|<frac13$ and $ sum^n_r=1a_rz^r = 1;,$ Where $|a_r|<2$Product of one minus the tenth roots of unityComplex number (cube roots of unity)Summation over roots of unityMaximum and Minimum of complex number $z$ where $|z^4+z^3-z-1| = |z^4-z^3+z-1|$.Proving that $sum cos p theta = 0$ where $theta$ is argument of the primitive roots of unityEvaluating the complex summationvalue of $|b_1|+|b_2|+cdots cdots +|b_k|$ isProve that $|z_1+z_2|^2leq (1+c)|z_1|^2+bigg(1+frac1cbigg)|z_2|^2$

How strictly should I take "Candidates must be local"?

What's the "normal" opposite of flautando?

They call me Inspector Morse

Distinction between apt-cache and dpkg -l

What are some noteworthy "mic-drop" moments in math?

Does a warlock using the Darkness/Devil's Sight combo still have advantage on ranged attacks against a target outside the Darkness?

Conservation of Mass and Energy

Error during using callback start_page_number in lualatex

Intuition behind counterexample of Euler's sum of powers conjecture

Bash script should only kill those instances of another script's that it has launched

PTIJ: wiping amalek’s memory?

How did Alan Turing break the enigma code using the hint given by the lady in the bar?

In the quantum hamiltonian, why does kinetic energy turn into an operator while potential doesn't?

Is "conspicuously missing" or "conspicuously" the subject of this sentence?

When traveling to Europe from North America, do I need to purchase a different power strip?

How can The Temple of Elementary Evil reliably protect itself against kinetic bombardment?

Recommendation letter by significant other if you worked with them professionally?

How do I express some one as a black person?

Accepted offer letter, position changed

Hotkey (or other quick way) to insert a keyframe for only one component of a vector-valued property?

Should I tell my boss the work he did was worthless

Are all players supposed to be able to see each others' character sheets?

In the late 1940’s to early 1950’s what technology was available that could melt a LOT of ice?

Declaring and defining template, and specialising them



double sigma in complex number


If complex no. ($z$) satisfying $frac12leq |z|leq 4;,$ Then Max. and Min. of $left|z+frac1zright|$complex no. $z$ such that $ |z|<frac13$ and $ sum^n_r=1a_rz^r = 1;,$ Where $|a_r|<2$Product of one minus the tenth roots of unityComplex number (cube roots of unity)Summation over roots of unityMaximum and Minimum of complex number $z$ where $|z^4+z^3-z-1| = |z^4-z^3+z-1|$.Proving that $sum cos p theta = 0$ where $theta$ is argument of the primitive roots of unityEvaluating the complex summationvalue of $|b_1|+|b_2|+cdots cdots +|b_k|$ isProve that $|z_1+z_2|^2leq (1+c)|z_1|^2+bigg(1+frac1cbigg)|z_2|^2$













1












$begingroup$


if $beta_1,beta_2,cdots ,beta_100$ are $100$ th roots of unity. The numerical value of $displaystyle mathopsum sum_1leq ileq jleq 100bigg(beta_ibeta_jbigg)^5$ is



what i try



assuming $x=(1)^frac1100,$ then $x^100-1=0$ has roots



$x=beta_1,beta_2,cdots,beta_100$



$displaystyle x^100-1=(x-beta_1)(x-beta_2)cdots cdots (x-beta_100)$



how do i solve it help me please










share|cite|improve this question









$endgroup$











  • $begingroup$
    This sum equals $1/2(sum_1leq i, j leq nbeta_i^5 beta_j^5 + sum_1 leq i leq nbeta_i ^ 5)$. Then use the lemma that $sum_i in S_1sum_jin S_2f(i)g(j) = (sum_i in S_1f(i)) cdot (sum_j in S_2 f(j))$
    $endgroup$
    – enedil
    Mar 7 at 14:51










  • $begingroup$
    @enedil dod not understand please explain me
    $endgroup$
    – jacky
    Mar 7 at 18:01






  • 1




    $begingroup$
    What did you not understand?
    $endgroup$
    – enedil
    Mar 7 at 19:50










  • $begingroup$
    @enedil first line
    $endgroup$
    – jacky
    Mar 7 at 20:20










  • $begingroup$
    First you double count elements, without checking the condition that $i leq j$, then add elements with indices $i, i$. Every element was doubly counted, so you just divide by two.
    $endgroup$
    – enedil
    Mar 7 at 20:32















1












$begingroup$


if $beta_1,beta_2,cdots ,beta_100$ are $100$ th roots of unity. The numerical value of $displaystyle mathopsum sum_1leq ileq jleq 100bigg(beta_ibeta_jbigg)^5$ is



what i try



assuming $x=(1)^frac1100,$ then $x^100-1=0$ has roots



$x=beta_1,beta_2,cdots,beta_100$



$displaystyle x^100-1=(x-beta_1)(x-beta_2)cdots cdots (x-beta_100)$



how do i solve it help me please










share|cite|improve this question









$endgroup$











  • $begingroup$
    This sum equals $1/2(sum_1leq i, j leq nbeta_i^5 beta_j^5 + sum_1 leq i leq nbeta_i ^ 5)$. Then use the lemma that $sum_i in S_1sum_jin S_2f(i)g(j) = (sum_i in S_1f(i)) cdot (sum_j in S_2 f(j))$
    $endgroup$
    – enedil
    Mar 7 at 14:51










  • $begingroup$
    @enedil dod not understand please explain me
    $endgroup$
    – jacky
    Mar 7 at 18:01






  • 1




    $begingroup$
    What did you not understand?
    $endgroup$
    – enedil
    Mar 7 at 19:50










  • $begingroup$
    @enedil first line
    $endgroup$
    – jacky
    Mar 7 at 20:20










  • $begingroup$
    First you double count elements, without checking the condition that $i leq j$, then add elements with indices $i, i$. Every element was doubly counted, so you just divide by two.
    $endgroup$
    – enedil
    Mar 7 at 20:32













1












1








1


1



$begingroup$


if $beta_1,beta_2,cdots ,beta_100$ are $100$ th roots of unity. The numerical value of $displaystyle mathopsum sum_1leq ileq jleq 100bigg(beta_ibeta_jbigg)^5$ is



what i try



assuming $x=(1)^frac1100,$ then $x^100-1=0$ has roots



$x=beta_1,beta_2,cdots,beta_100$



$displaystyle x^100-1=(x-beta_1)(x-beta_2)cdots cdots (x-beta_100)$



how do i solve it help me please










share|cite|improve this question









$endgroup$




if $beta_1,beta_2,cdots ,beta_100$ are $100$ th roots of unity. The numerical value of $displaystyle mathopsum sum_1leq ileq jleq 100bigg(beta_ibeta_jbigg)^5$ is



what i try



assuming $x=(1)^frac1100,$ then $x^100-1=0$ has roots



$x=beta_1,beta_2,cdots,beta_100$



$displaystyle x^100-1=(x-beta_1)(x-beta_2)cdots cdots (x-beta_100)$



how do i solve it help me please







complex-numbers






share|cite|improve this question













share|cite|improve this question











share|cite|improve this question




share|cite|improve this question










asked Mar 7 at 14:36









jackyjacky

1,028715




1,028715











  • $begingroup$
    This sum equals $1/2(sum_1leq i, j leq nbeta_i^5 beta_j^5 + sum_1 leq i leq nbeta_i ^ 5)$. Then use the lemma that $sum_i in S_1sum_jin S_2f(i)g(j) = (sum_i in S_1f(i)) cdot (sum_j in S_2 f(j))$
    $endgroup$
    – enedil
    Mar 7 at 14:51










  • $begingroup$
    @enedil dod not understand please explain me
    $endgroup$
    – jacky
    Mar 7 at 18:01






  • 1




    $begingroup$
    What did you not understand?
    $endgroup$
    – enedil
    Mar 7 at 19:50










  • $begingroup$
    @enedil first line
    $endgroup$
    – jacky
    Mar 7 at 20:20










  • $begingroup$
    First you double count elements, without checking the condition that $i leq j$, then add elements with indices $i, i$. Every element was doubly counted, so you just divide by two.
    $endgroup$
    – enedil
    Mar 7 at 20:32
















  • $begingroup$
    This sum equals $1/2(sum_1leq i, j leq nbeta_i^5 beta_j^5 + sum_1 leq i leq nbeta_i ^ 5)$. Then use the lemma that $sum_i in S_1sum_jin S_2f(i)g(j) = (sum_i in S_1f(i)) cdot (sum_j in S_2 f(j))$
    $endgroup$
    – enedil
    Mar 7 at 14:51










  • $begingroup$
    @enedil dod not understand please explain me
    $endgroup$
    – jacky
    Mar 7 at 18:01






  • 1




    $begingroup$
    What did you not understand?
    $endgroup$
    – enedil
    Mar 7 at 19:50










  • $begingroup$
    @enedil first line
    $endgroup$
    – jacky
    Mar 7 at 20:20










  • $begingroup$
    First you double count elements, without checking the condition that $i leq j$, then add elements with indices $i, i$. Every element was doubly counted, so you just divide by two.
    $endgroup$
    – enedil
    Mar 7 at 20:32















$begingroup$
This sum equals $1/2(sum_1leq i, j leq nbeta_i^5 beta_j^5 + sum_1 leq i leq nbeta_i ^ 5)$. Then use the lemma that $sum_i in S_1sum_jin S_2f(i)g(j) = (sum_i in S_1f(i)) cdot (sum_j in S_2 f(j))$
$endgroup$
– enedil
Mar 7 at 14:51




$begingroup$
This sum equals $1/2(sum_1leq i, j leq nbeta_i^5 beta_j^5 + sum_1 leq i leq nbeta_i ^ 5)$. Then use the lemma that $sum_i in S_1sum_jin S_2f(i)g(j) = (sum_i in S_1f(i)) cdot (sum_j in S_2 f(j))$
$endgroup$
– enedil
Mar 7 at 14:51












$begingroup$
@enedil dod not understand please explain me
$endgroup$
– jacky
Mar 7 at 18:01




$begingroup$
@enedil dod not understand please explain me
$endgroup$
– jacky
Mar 7 at 18:01




1




1




$begingroup$
What did you not understand?
$endgroup$
– enedil
Mar 7 at 19:50




$begingroup$
What did you not understand?
$endgroup$
– enedil
Mar 7 at 19:50












$begingroup$
@enedil first line
$endgroup$
– jacky
Mar 7 at 20:20




$begingroup$
@enedil first line
$endgroup$
– jacky
Mar 7 at 20:20












$begingroup$
First you double count elements, without checking the condition that $i leq j$, then add elements with indices $i, i$. Every element was doubly counted, so you just divide by two.
$endgroup$
– enedil
Mar 7 at 20:32




$begingroup$
First you double count elements, without checking the condition that $i leq j$, then add elements with indices $i, i$. Every element was doubly counted, so you just divide by two.
$endgroup$
– enedil
Mar 7 at 20:32










1 Answer
1






active

oldest

votes


















1












$begingroup$


We obtain
beginalign*
colorbluesum_1leq ileq jleq 100colorblueleft(beta_ibeta_jright)^5
&=sum_j=1^100beta_j^5sum_i=1^jbeta_i^5tag1\
&=sum_j=1^100beta_1^5jsum_i=1^jleft(beta_1^5right)^itag2\
&=sum_j=1^100beta_1^5jfracbeta_1^5-left(beta_1^5right)^j+11-beta_1tag3\
&=fracbeta_1^51-beta_1left(sum_j=1^100beta_1^5j-sum_j=1^100beta_1^10jright)tag4\
&=fracbeta_1^51-beta_1left(fracbeta_1^5-beta_1^5051-beta_1^5-fracbeta_1^10-beta_1^10101-beta_1^10right)tag5\
&,,colorblue=0
endalign*




Comment:



  • In (1) we rewrite the sum as double sum and factor out $beta_j$.


  • In (2) we use $beta_j=expleft(frac2pi i j100right)=expleft(frac2pi i100right)^j=beta_1^j$.


  • In (3) we apply the finite geometric series formula.


  • In (4) we factor out common terms and multiply out the sums.


  • In (5) we again apply the finite geometric series formula. We observe, since $beta_1^100=1$ that $beta_1^505=beta_1^5$ and $beta_1^10=beta_1^1010$.


Note:



Since we have $beta_1^100-1=(beta_1-1)left(1+beta_1+cdots+beta_1^99right)=0$ we can deduce the result zero already from (4).






share|cite|improve this answer









$endgroup$












    Your Answer





    StackExchange.ifUsing("editor", function ()
    return StackExchange.using("mathjaxEditing", function ()
    StackExchange.MarkdownEditor.creationCallbacks.add(function (editor, postfix)
    StackExchange.mathjaxEditing.prepareWmdForMathJax(editor, postfix, [["$", "$"], ["\\(","\\)"]]);
    );
    );
    , "mathjax-editing");

    StackExchange.ready(function()
    var channelOptions =
    tags: "".split(" "),
    id: "69"
    ;
    initTagRenderer("".split(" "), "".split(" "), channelOptions);

    StackExchange.using("externalEditor", function()
    // Have to fire editor after snippets, if snippets enabled
    if (StackExchange.settings.snippets.snippetsEnabled)
    StackExchange.using("snippets", function()
    createEditor();
    );

    else
    createEditor();

    );

    function createEditor()
    StackExchange.prepareEditor(
    heartbeatType: 'answer',
    autoActivateHeartbeat: false,
    convertImagesToLinks: true,
    noModals: true,
    showLowRepImageUploadWarning: true,
    reputationToPostImages: 10,
    bindNavPrevention: true,
    postfix: "",
    imageUploader:
    brandingHtml: "Powered by u003ca class="icon-imgur-white" href="https://imgur.com/"u003eu003c/au003e",
    contentPolicyHtml: "User contributions licensed under u003ca href="https://creativecommons.org/licenses/by-sa/3.0/"u003ecc by-sa 3.0 with attribution requiredu003c/au003e u003ca href="https://stackoverflow.com/legal/content-policy"u003e(content policy)u003c/au003e",
    allowUrls: true
    ,
    noCode: true, onDemand: true,
    discardSelector: ".discard-answer"
    ,immediatelyShowMarkdownHelp:true
    );



    );













    draft saved

    draft discarded


















    StackExchange.ready(
    function ()
    StackExchange.openid.initPostLogin('.new-post-login', 'https%3a%2f%2fmath.stackexchange.com%2fquestions%2f3138939%2fdouble-sigma-in-complex-number%23new-answer', 'question_page');

    );

    Post as a guest















    Required, but never shown

























    1 Answer
    1






    active

    oldest

    votes








    1 Answer
    1






    active

    oldest

    votes









    active

    oldest

    votes






    active

    oldest

    votes









    1












    $begingroup$


    We obtain
    beginalign*
    colorbluesum_1leq ileq jleq 100colorblueleft(beta_ibeta_jright)^5
    &=sum_j=1^100beta_j^5sum_i=1^jbeta_i^5tag1\
    &=sum_j=1^100beta_1^5jsum_i=1^jleft(beta_1^5right)^itag2\
    &=sum_j=1^100beta_1^5jfracbeta_1^5-left(beta_1^5right)^j+11-beta_1tag3\
    &=fracbeta_1^51-beta_1left(sum_j=1^100beta_1^5j-sum_j=1^100beta_1^10jright)tag4\
    &=fracbeta_1^51-beta_1left(fracbeta_1^5-beta_1^5051-beta_1^5-fracbeta_1^10-beta_1^10101-beta_1^10right)tag5\
    &,,colorblue=0
    endalign*




    Comment:



    • In (1) we rewrite the sum as double sum and factor out $beta_j$.


    • In (2) we use $beta_j=expleft(frac2pi i j100right)=expleft(frac2pi i100right)^j=beta_1^j$.


    • In (3) we apply the finite geometric series formula.


    • In (4) we factor out common terms and multiply out the sums.


    • In (5) we again apply the finite geometric series formula. We observe, since $beta_1^100=1$ that $beta_1^505=beta_1^5$ and $beta_1^10=beta_1^1010$.


    Note:



    Since we have $beta_1^100-1=(beta_1-1)left(1+beta_1+cdots+beta_1^99right)=0$ we can deduce the result zero already from (4).






    share|cite|improve this answer









    $endgroup$

















      1












      $begingroup$


      We obtain
      beginalign*
      colorbluesum_1leq ileq jleq 100colorblueleft(beta_ibeta_jright)^5
      &=sum_j=1^100beta_j^5sum_i=1^jbeta_i^5tag1\
      &=sum_j=1^100beta_1^5jsum_i=1^jleft(beta_1^5right)^itag2\
      &=sum_j=1^100beta_1^5jfracbeta_1^5-left(beta_1^5right)^j+11-beta_1tag3\
      &=fracbeta_1^51-beta_1left(sum_j=1^100beta_1^5j-sum_j=1^100beta_1^10jright)tag4\
      &=fracbeta_1^51-beta_1left(fracbeta_1^5-beta_1^5051-beta_1^5-fracbeta_1^10-beta_1^10101-beta_1^10right)tag5\
      &,,colorblue=0
      endalign*




      Comment:



      • In (1) we rewrite the sum as double sum and factor out $beta_j$.


      • In (2) we use $beta_j=expleft(frac2pi i j100right)=expleft(frac2pi i100right)^j=beta_1^j$.


      • In (3) we apply the finite geometric series formula.


      • In (4) we factor out common terms and multiply out the sums.


      • In (5) we again apply the finite geometric series formula. We observe, since $beta_1^100=1$ that $beta_1^505=beta_1^5$ and $beta_1^10=beta_1^1010$.


      Note:



      Since we have $beta_1^100-1=(beta_1-1)left(1+beta_1+cdots+beta_1^99right)=0$ we can deduce the result zero already from (4).






      share|cite|improve this answer









      $endgroup$















        1












        1








        1





        $begingroup$


        We obtain
        beginalign*
        colorbluesum_1leq ileq jleq 100colorblueleft(beta_ibeta_jright)^5
        &=sum_j=1^100beta_j^5sum_i=1^jbeta_i^5tag1\
        &=sum_j=1^100beta_1^5jsum_i=1^jleft(beta_1^5right)^itag2\
        &=sum_j=1^100beta_1^5jfracbeta_1^5-left(beta_1^5right)^j+11-beta_1tag3\
        &=fracbeta_1^51-beta_1left(sum_j=1^100beta_1^5j-sum_j=1^100beta_1^10jright)tag4\
        &=fracbeta_1^51-beta_1left(fracbeta_1^5-beta_1^5051-beta_1^5-fracbeta_1^10-beta_1^10101-beta_1^10right)tag5\
        &,,colorblue=0
        endalign*




        Comment:



        • In (1) we rewrite the sum as double sum and factor out $beta_j$.


        • In (2) we use $beta_j=expleft(frac2pi i j100right)=expleft(frac2pi i100right)^j=beta_1^j$.


        • In (3) we apply the finite geometric series formula.


        • In (4) we factor out common terms and multiply out the sums.


        • In (5) we again apply the finite geometric series formula. We observe, since $beta_1^100=1$ that $beta_1^505=beta_1^5$ and $beta_1^10=beta_1^1010$.


        Note:



        Since we have $beta_1^100-1=(beta_1-1)left(1+beta_1+cdots+beta_1^99right)=0$ we can deduce the result zero already from (4).






        share|cite|improve this answer









        $endgroup$




        We obtain
        beginalign*
        colorbluesum_1leq ileq jleq 100colorblueleft(beta_ibeta_jright)^5
        &=sum_j=1^100beta_j^5sum_i=1^jbeta_i^5tag1\
        &=sum_j=1^100beta_1^5jsum_i=1^jleft(beta_1^5right)^itag2\
        &=sum_j=1^100beta_1^5jfracbeta_1^5-left(beta_1^5right)^j+11-beta_1tag3\
        &=fracbeta_1^51-beta_1left(sum_j=1^100beta_1^5j-sum_j=1^100beta_1^10jright)tag4\
        &=fracbeta_1^51-beta_1left(fracbeta_1^5-beta_1^5051-beta_1^5-fracbeta_1^10-beta_1^10101-beta_1^10right)tag5\
        &,,colorblue=0
        endalign*




        Comment:



        • In (1) we rewrite the sum as double sum and factor out $beta_j$.


        • In (2) we use $beta_j=expleft(frac2pi i j100right)=expleft(frac2pi i100right)^j=beta_1^j$.


        • In (3) we apply the finite geometric series formula.


        • In (4) we factor out common terms and multiply out the sums.


        • In (5) we again apply the finite geometric series formula. We observe, since $beta_1^100=1$ that $beta_1^505=beta_1^5$ and $beta_1^10=beta_1^1010$.


        Note:



        Since we have $beta_1^100-1=(beta_1-1)left(1+beta_1+cdots+beta_1^99right)=0$ we can deduce the result zero already from (4).







        share|cite|improve this answer












        share|cite|improve this answer



        share|cite|improve this answer










        answered 2 days ago









        Markus ScheuerMarkus Scheuer

        62.6k459149




        62.6k459149



























            draft saved

            draft discarded
















































            Thanks for contributing an answer to Mathematics Stack Exchange!


            • Please be sure to answer the question. Provide details and share your research!

            But avoid


            • Asking for help, clarification, or responding to other answers.

            • Making statements based on opinion; back them up with references or personal experience.

            Use MathJax to format equations. MathJax reference.


            To learn more, see our tips on writing great answers.




            draft saved


            draft discarded














            StackExchange.ready(
            function ()
            StackExchange.openid.initPostLogin('.new-post-login', 'https%3a%2f%2fmath.stackexchange.com%2fquestions%2f3138939%2fdouble-sigma-in-complex-number%23new-answer', 'question_page');

            );

            Post as a guest















            Required, but never shown





















































            Required, but never shown














            Required, but never shown












            Required, but never shown







            Required, but never shown

































            Required, but never shown














            Required, but never shown












            Required, but never shown







            Required, but never shown







            Popular posts from this blog

            Lowndes Grove History Architecture References Navigation menu32°48′6″N 79°57′58″W / 32.80167°N 79.96611°W / 32.80167; -79.9661132°48′6″N 79°57′58″W / 32.80167°N 79.96611°W / 32.80167; -79.9661178002500"National Register Information System"Historic houses of South Carolina"Lowndes Grove""+32° 48' 6.00", −79° 57' 58.00""Lowndes Grove, Charleston County (260 St. Margaret St., Charleston)""Lowndes Grove"The Charleston ExpositionIt Happened in South Carolina"Lowndes Grove (House), Saint Margaret Street & Sixth Avenue, Charleston, Charleston County, SC(Photographs)"Plantations of the Carolina Low Countrye

            random experiment with two different functions on unit interval Announcing the arrival of Valued Associate #679: Cesar Manara Planned maintenance scheduled April 23, 2019 at 00:00UTC (8:00pm US/Eastern)Random variable and probability space notionsRandom Walk with EdgesFinding functions where the increase over a random interval is Poisson distributedNumber of days until dayCan an observed event in fact be of zero probability?Unit random processmodels of coins and uniform distributionHow to get the number of successes given $n$ trials , probability $P$ and a random variable $X$Absorbing Markov chain in a computer. Is “almost every” turned into always convergence in computer executions?Stopped random walk is not uniformly integrable

            How should I support this large drywall patch? Planned maintenance scheduled April 23, 2019 at 00:00UTC (8:00pm US/Eastern) Announcing the arrival of Valued Associate #679: Cesar Manara Unicorn Meta Zoo #1: Why another podcast?How do I cover large gaps in drywall?How do I keep drywall around a patch from crumbling?Can I glue a second layer of drywall?How to patch long strip on drywall?Large drywall patch: how to avoid bulging seams?Drywall Mesh Patch vs. Bulge? To remove or not to remove?How to fix this drywall job?Prep drywall before backsplashWhat's the best way to fix this horrible drywall patch job?Drywall patching using 3M Patch Plus Primer